Two lakes in the Pawpaw mountains, Quapaw and Highwater, were suffering from serious declines in their fish populatio...

JayDee8732 on July 6, 2017

Why is B the answer

I need help on this one why is B the answer and why is A incorrect?

Reply
Create a free account to read and take part in forum discussions.

Already have an account? log in

Mehran on July 10, 2017

@JayDee8732 this is a Cause & Effect argument.

The observed effect the author is trying to explain? The rebound in the fish population at Quapaw Lake.

The author's proposed cause? The ban on fishing, i.e. less fishing.

This is a Weaken question, so we are looking for an answer choice that weakens this argument.

How do we weaken a Cause & Effect argument?

(1) Cause without Effect

(2) Effect without Cause

(3) Alternate Cause

(A) is completely irrelevant here. This argument is about the reason for the rebound in fish at Quapaw Lake. This argument is not about the reason for the continued decline of the fish population at Highwater Lake.

(B) is the correct answer because it effectively shows effect (i.e. rebound) without cause (i.e. less fishing). There was never a fishing issue at Quapaw Lake, so the fishing ban would not have impacted the amount of fishing taking place.

As such, (B) clearly weakens the argument and it is the correct answer.

Hope that helps! Please let us know if you have any other questions.